i will mark branlyest
Jess dug a hole 7.3 inches deep on Tuesday. After he dug on Wednesday, the hole was 9.7 inches deep. What distance did Jess dig on Wednesday? A –17 inches B –2.4 inches C 2.4 inches D 17 inches

Answers

Answer 1

Answer:

Jess dug 2.4 inches deep on Wednesday.

Step-by-step explanation:

9.7-7.3=2.4 9 . 7

-

7 . 3

___

2 . 4

Answer 2

Answer:

2.4 inches

Step-by-step explanation:

i did the assignment :)


Related Questions

Please help me with this

Answers

Answer:

x = 5

Step-by-step explanation:

Given

- 3x - 2 = - 17 ( add 2 to both sides )

- 3x = - 15 ( divide both sides by - 3 )

x = 5

Help me please!!!!!!!!!!!!!!

Answers

Answer: reflection over the y axis

Step-by-step explanation:

Solve for x. Use the photo for the chord

Answers

Answer: x=3

Step-by-step explanation:

Answer and explanation

Answers

Answer:

392in

Step-by-step explanation:

find the areas of all the sides of this shape. Of one you might have to split it into two shpaes.

Solve the quadratic equation by using either numeric or graphic approach x^2 + 2x - 3 = 0

Answers

Answer:

d

Step-by-step explanation:

[tex]x 1= \frac{ - b + \sqrt{ {b}^{2} - 4ac} }{2a} \\ x2 = \frac{ - b - \sqrt{ {b}^{2} - 4ac} }{2a} \\ x1 = \frac{ - 2 + \sqrt{ {2}^{2} - 4\times 1 \times ( - 3)} }{2} = 1 \\ x2 = \frac{ - 2 - \sqrt{ {2}^{2} - 4 \times 1 \times ( - 3)} }{2} = - 3[/tex]

Answer:

d

Step-by-step explanation:

Solving by factorising

Given

x² + 2x - 3 = 0 ← in standard form

(x + 3)(x - 1) = 0 ← in factored form

Equate each factor to zero and solve for x

x + 3 = 0 ⇒ x = - 3

x - 1 = 0 ⇒ x = 1

which is the correct input-output table for the function g(x)=-1/2(4x+6)

Answers

Answer: The third choice.

Step-by-step explanation: To find the first output, start with your first input. In this case, it's -2, so your equation would be g(-2)= -1/2(4(-2)+6). Start by solving inside the parenthesis. 4x-2=-8 and -8+6=-2. -1/2x-2= 1, so your first output should be 1. The only choice that has this is the third one, so that is your answer. Hope I could help :)

Pisces is replacing paving stones for her inground pool. Her pool is 10m by 5m and is surrounded by a 1 1/2 border of paving stone. If each paving stone was 25 cm by 40 cm, in theory, how many paving stones will she need?

Answers

Answer:

The number of paving stones she needs is 540 paving stone

Step-by-step explanation:

The given dimensions of Pisces' inground pool = 10 m by 5 m

The thickness of the border surrounding the pool = 1 1/2

The dimensions of each paving stone = 25 cm by 40 cm = 0.25 m by 0.4 m

The length dimension of the pool and border = 10 + 2 × 1 1/2 = 13 m

The width dimension of the pool = 5 + 2 × 1 1/2 = 8 m

The total area of the paving stone, [tex]T_A[/tex] = Area of pool + border - Area of pool

∴ [tex]T_A[/tex] = 13 m × 8 m - 10 m × 5 m = 54 m²

The total area of the paving stone = 54 m²

The area of each paving stone, [tex]A_{EP}[/tex] = 0.25 m × 0.4 m = 0.1 m²/paving stone

The number of paving stones she needs, n = (The total area of the paving stone)/(The area of each paving stone)

∴ n = 54 m²/(0.1 m²/paving stone) = 540 paving stone.

Diana finished 5/8 of her homework. what percentage is equivalent to the fraction of homework that Diana completed

Answers

Answer:62.5%

Step-by-step explanation:

A rocket is launched from a tower. The height of the rocket, y in feet, is related to the time after launch, x in seconds, by the given equation. Using this equation, find the time that the rocket will hit the ground, to the nearest 100th of second.(Quadratic Word Problems (Profit/Gravity)
y=-16x^2+121x+83

Answers

Answer:

The rocket will hit the ground after about 8.20 seconds.

Step-by-step explanation:

The height of the rocket y, in feet, x seconds after launch is modeled by the equation:

[tex]y=-16x^2+121x+83[/tex]

We want to find the time at which the rocket will hit the ground.

If it hits the ground, the height of the rocket y will be 0. Thus:

[tex]0=-16x^2+121x+83[/tex]

We can solve for x. Factoring (if possible at all) or completing the square can be tedious, so we can use the quadratic formula:

[tex]\displaystyle x=\frac{-b\pm\sqrt{b^2-4ac}}{2a}[/tex]

In this case, a = -16, b = 121, and c = 83. Substitute:

[tex]\displaystyle x=\frac{-(121)\pm\sqrt{(121)^2-4(-16)(83)}}{2(-16)}[/tex]

Simplify:

[tex]\displaystyle x=\frac{-121\pm\sqrt{19953}}{-32}[/tex]

Divide everything by -1 and simplify the square root. The plus/minus will remain unchanged:

[tex]\displaystyle x=\frac{121\pm3\sqrt{2217}}{32}[/tex]

Therefore, our two solutions are:

[tex]\displaystyle x=\frac{121+3\sqrt{2217}}{32}\approx 8.20\text{ or } x=\frac{121-3\sqrt{2217}}{32}\approx -0.63[/tex]

Since time cannot be negative, we can ignore the second solution.

Therefore, the rocket will hit the ground after about 8.20 seconds.

Please show all work and first right answer gets brainly.

Answers

Answer:

im not good with math sorry :(

Step-by-step explanation:


Note Figure is not drawn to scale.
If r= 2 cm and h = 2 cm, what is the surface area,
in terms of r?

Answers

Answer:

your answer is...

Step-by-step explanation:

50.265482457437

I used my calculator

what is the difference between rational or irrational numbers

Answers

Answer:

In rational numbers, both numerator and denominator are whole numbers, where the denominator is not equal to zero. A irrational number is a number which cannot be expressed in a ratio of two integers

Step-by-step explanation:


PLEASE HELP!!!
This graph shows the relationship between demand and price for goods sold at a shop. Anna plots the data on a scatter plot and draws a line of best fit to represent the relationship as shown.

This is not the line of best fit because the line ____________.
A.) Is close to the data points
B.) does not fit every data point exactly.
C.) Is not through the middle of the data points.

Anna could get a better fit by _____
A.) Decreasing
B.) Increasing
C.) Keeping

the y-intercept and ______ the slope.
A.) Decreasing
B.) Increasing
C.) Keeping

Answers

Answer:

1st question:  C

2nd question: Increasing (the slope)

3rd question:  don't understand what's being asked

Step-by-step explanation:

Answer:

the answers are

Step-bythe answers are -step explanation:

1.C

2.a

3.B

Liam's baking plans changed. He
ended up baking 33 cookies. He
kept 5 for himself and split the
rest evenly between 7 friends.
H
What will the new equation be?

Answers

Answer:

(33 - 5)/7 = 4

Step-by-step explanation:

Border fencing costs $2.89 per package, and each package will fence a length of 8 feet.

What will the cost be to buy fencing to go around the triangular flower bed?

$49.13
$23.12
$390.15

Answers

Answer:

$23.12

Step-by-step explanation:

you'll need 3 sides so take 8×2.89

answer is $23.12

It would be 23.12 that should be the answer

Triangle ABC has the following vertices:
· A(1,9)
B(11, -7)
C(-9,3)
Is triangle ABC a right triangle, and why?

Answers

Answer:

yes

Step-by-step explanation:

Answer:

No,

Step-by-step explanation:because △ABC doesn't have a pair of perpendicular sides.

Multiple Choice Question Find the y-value when x = 0 in the equation y= 2x + 3. A. 5 B. 2 C. 3 D. 1​

Answers

Answer:  C.

brainliest plz

Step-by-step explanation:

y=2x+3

substitute in x

y=(2*0)+3

y=0+3

y=3

Ethan swam 3.6 miles on Monday and 2.75 miles on Tuesday. How many total miles did Ethan swim on both days?

Answers

Answer:

6.35

Step-by-step explanation:

you have to add 3.6 + 2.75

Answer:

6.35 miles (give brainliest)

Step-by-step explanation:

Hello! Sorry about My last question! I was lying this is just a Homework assignment and it is completion Credit but if you do it I will still give you the brainliest!

Answers

To find the percent of something, turn the percent into a decimal and multiply (move the decimal place 2 places left, e.g. 53% = .53)

it should be 53 percent or 0.53 :))

3 3/4 x 4

Help me pas

Answers

Convert any mixed numbers to fractions.
Then your initial equation becomes:

15
4
×
4
1
Applying the fractions formula for multiplication,

=
15
×
4
4
×
1

=
60
4
Simplifying 60/4, the answer is
15

Answer:

15 (C)

//msg added cus it was short//

What’s the answer for TS? (Arc measure)

NO LINKS!

Answers

angle TS = 74°

Straight line is 180°, so add PT and SR

(64+42 = 106)

Subtract 180 from 106, you get 74 :)

Plzzzz asappp help thanks

Answers

Answer:

I might not be right but try following the steps in the screen shot

Also the answer is in the screenshot

Step-by-step explanation:

I tried my best. :>

Please help!! I need answering

Answers

Answer:

B

Step-by-step explanation:

26x82=2132

2132/2=1066

80% of a number is x. What is 100% of the number? Assume x > 0.​

Answers

Answer:

Here's a calculator  link hope it helps : )

Step-by-step explanation:

https://www.mathpapa.com/algebra-calculator.html

What are the measurements of FGT plssss?​

Answers

Answer:

The Foster–Greer–Thorbecke indices are a family of poverty metrics. The most commonly used index from the family, FGT2, puts higher weight on the poverty of the poorest individuals, making it a combined measure of poverty and income inequality and a popular choice within development economics.

Step-by-step explanation:

0000000

What value of x makes the equation true?
3 - 14x = 17
A-1
B-1
C-28
D-2

Answers

Answer:

3 -14x =17

-14x = 17 - 3

-14x = 14

x = 14/-14 = 1/-1 = -1

∴ answer = -1

hope this answer helps you.....

please mark as brainliest... thank you!

I think it’s an ac130 or an a10

A laptop carry bag cost $30. The total cost will include an 8% sales tax. Which expression can be used in calculating the total cost with tax?

A. 0.8(30)

B. 1.08 (30)

C. 0.08 (30)

D. 1.8 (30)

Answers

The correct answer to this is C, you’re welcome
The answer is C because you are calculating for a percentage

Mrs Fuller plants a 4 inch tomato plant in her garden. The tomato plant grows at a rate of 2 inches each week. Which equation can be used to determine h, the height of the tomato plant in inches after w weeks?

Answers

Answer: 4 + 2w

Step-by-step explanation:

The height of the tomato plant in inches after w weeks will be the value of the current height plus the weekly growth pee week. This can be represented in an equation as:

h = 4 + 2(w)

h = 4 + 2w

For example, the height of the tomato plant in inches after 5 weeks will be:

h = 4 + 2w

h = 4 + 2(5)

h = 4 + 10

h = 14 inches

Find the circumference of the circle.
A. 11 cm
B. 44 cm
C. 22 cm
D. 33 cm

Answers

Answer:

The answer is 44 cm .

which is (B)


PLEASE HEEEEEELP!!

Answers

Answer:

1 is 105°

2 is 75°

Step-by-step explanation:

75° and angle 2 are corresponding angles(equal)

angles on straight line sum up to 180°

let x be angle 1

x+ 75= 180

x =180-75

x= 105°

Answer:

1. is 105

2. 75 degrees

Step-by-step explanation:

Other Questions
Help me pls-Describe Jays Treaty. Be detailed. Who was president when it was signed and which countries signed it? (11/30) Why is the windmill so important in the novel Animal Farm? What is the value of the expression 1 2 3 4 5? Is Gilgamesh in love with Enkidu? When I was walking down the street, I saw a womanman in the face.O a) to hitOb) hittingOc) hitO d) was hitting She ___ her wallet in the internet caf, but I am not sure Is quadratic equation class 11? Is area and volume inversely proportional? how is the idealism of youth a big idea in the three musketeers? Why does the speaker say we do not need the wall Mending Wall? Suppose previous research suggests that the mean length of all adult Anacondas is 13 feet with a standard deviation of 2.4 feet. Let W be the random variable that stands for length of adult Anacondas, so E(W)=13, SD(W)=2.4. You are planning on collecting a random sample of 50 adult Anacondas. Consider the RV Bar-W, which is the mean of the 50 sampled Anacondas. 98% of samples will have the realized value of Bar-W less than what value? Which of the answers reasonably approximates the requested value of the sample mean with justification?a. "Bar-w"=14.67 since we can use a normal approximation by the CLTb. "Bar-w"=17.93 since we can use a normal approximation by the CLTc. "Bar-w"=13.70 since we can use a normal approximation by the CLTd. "Bar-w"=12.30 since we can use a normal approximation by the CLTe.A normal approximation is inappropriate After having seen many pictures of the Lincoln Monument during his lifetime, Mr. Adamsmistakenly recalled that he had actually visited the site. This best illustratesanswer choices:source amnesia.proactive interference.the self-reference effect.flashbulb memory. How did Newspaper affect US support for military action in Cuba? Suppose two dice are rolled. Let X be the random variable measuring the sum of the two numbers rolled.(a) Find the probability mass function for X.(b) Find the expected value E(X).(c) Find the variance V(X). 5. True/False - Meanings are in words, not in people.a) Trueb) False6. _______refers to the believability of the speaker.a) credibility; b) charisma; c) character; d) chivalry7. True/False - The ethical pyramid is a way of understanding the three parts of ethics: intent, means and ends.a) Trueb) False Why do you think it is important for business to come up with new product development? Select all that apply which of the following is true of strophic form? multiple select question. it is almost always used in folksongs. it has minimal piano accompaniment. it makes a song easier to remember. it repeats the same music for each stanza. What are examples of dependent and independent? during which step of the personal-selling process does the salesperson establish a rapport that sets a foundation for the relationship and asks questions to learn more about the prospect and his or her needs and wants? Which type of reaction will this equation be classified as?